Đến nội dung

Kool LL nội dung

Có 363 mục bởi Kool LL (Tìm giới hạn từ 10-06-2020)



Sắp theo                Sắp xếp  

#433946 [TSĐH 2013] Đề thi môn toán khối D

Đã gửi bởi Kool LL on 09-07-2013 - 10:46 trong Thi TS ĐH





đề thi toán khối D

Câu 2: $sin3x + cos2x - sinx = 0$

$\Leftrightarrow 2cos2xsinx + cos2x = 0$

$\Leftrightarrow cos2x = 0 \vee sinx = \frac{-1}{2}$

$\Leftrightarrow x=\frac{\pi}{4} + \frac{k\pi}{2} \vee x=-\frac{\pi}{6}+l2\pi \vee x=\frac{7\pi}{6}+m2\pi$

$(k, l, m \in \mathbb{Z})$




#433962 [TSĐH 2013] Đề thi môn toán khối D

Đã gửi bởi Kool LL on 09-07-2013 - 11:47 trong Thi TS ĐH

Câu 3.

$2\log_2x+\log_{\frac{1}{2}}(1-\sqrt{x})=\frac{1}{2}\log_{\sqrt{2}}(x – 2\sqrt{x}+2).$ ( ĐK: $0< x < 1$ )
$\Leftrightarrow \log_2x^2-\log_2(1-\sqrt{x})=\log_2(x – 2\sqrt{x}+2)$

$\Leftrightarrow \log_2\frac{x^2}{1-\sqrt{x}}=\log_2(x – 2\sqrt{x}+2)$

$\Leftrightarrow \frac{x^2}{1-\sqrt{x}}=x – 2\sqrt{x}+2.$    $(*)$

Đặt $t=\sqrt{x}$ ( ĐK: $0<t<1$ ), pt (*) trở thành :

$\frac{t^4}{1-t}=t^2 – 2t+2$

$\Leftrightarrow t^4+t^3-3t^2+4t-2=0$

$\Leftrightarrow (t^2-t+1)(t^2-2t-2)=0$

$\Leftrightarrow t^2-2T-2=0$   ( do pt $t^2-t+1=0$ vô nghiệm )

$\Leftrightarrow t=-1-\sqrt{3}$ (loại)  $\vee t=-1+\sqrt{3}$ (nhận)

Vậy pt chỉ có duy nhất một nghiệm là $x=t^2=(-1+\sqrt{3})^2=4-2\sqrt{3}.$




#433970 [TSĐH 2013] Đề thi môn toán khối D

Đã gửi bởi Kool LL on 09-07-2013 - 12:05 trong Thi TS ĐH

Câu 4 (1,0 điểm). Tính tích phân

$I = \int\limits_0^1 \frac{(x+1)^2}{x^2+1}dx=\int\limits_0^1dx+\int\limits_0^1\frac{2x}{x^2+1}dx=x|_0^1+\int\limits_0^1\frac{d(x^2+1)}{x^2+1}$

   $=x|_0^1 + \ln|x^2+1||_0^1=1+\ln{2}$

 

 




#433982 [TSĐH 2013] Đề thi môn toán khối B

Đã gửi bởi Kool LL on 09-07-2013 - 12:55 trong Thi TS ĐH

$y=2x^3-3(m+1)x^2+6mx$.   (1)

$(d) : y=x+2$

$y' = 6{x^2} - 6(m + 1)x + 6m=6[x^2 - (m+1)x+m]$
* Đồ thị hs (1) có 2 điểm cực trị A,B nên $\Delta  = (m + 1)^2-4m=(m-1)^2 > 0 \Leftrightarrow m \ne 1$

*$y=\frac{1}{6}y'.(2x-m-1)-(m-1)^2x+(m^2+m)$
$\Rightarrow$ Pt đường thẳng qua 2 điểm cực trị là $(AB): y = -(m-1)^2x + (m^2 + m)$
*$(AB) \perp (d) \Leftrightarrow -(m-1)^2.1 = -1 \Leftrightarrow m = 0$ (nhận) $\vee  m = 2$ (nhận)

Vậy m thoả ycbt $\Leftrightarrow m=0 \vee m=2$.




#434005 [TSĐH 2013] Đề thi môn toán khối B

Đã gửi bởi Kool LL on 09-07-2013 - 15:26 trong Thi TS ĐH

Câu 6 (1,0 điểm). Cho $a,b,c$ là các số thực dương. Tìm giá trị lớn nhất của biểu thức:
$$P = \frac{4}{\sqrt{a^2+b^2+c^2-4}}-\frac{9}{(a+b)\sqrt{(a+2c)(b+2c)}}$$

Giải : Dùng AM-GM đánh giá được :

  • $a^2+b^2+c^2\ge \frac{(a+b+c)^2}{3}$
  • $(a+b)\sqrt{(a+2c)(b+2c)}\le (a+b) \left(\frac{a+b+4c}{2}\right)=\frac{1}{6}(3a+3b)(a+b+4c)$ $ \le \frac{1}{6} \left( \frac{4a+4b+4c}{2}\right)^2=\frac{2}{3}(a+b+c)^2$
  • Tất cả các dấu "=" xảy ra khi và chỉ khi $a=b=c$.

Suy ra : $P\le \frac{4}{ \sqrt{ \frac{(a+b+c)^2}{3}+4}}-\frac{27}{2(a+b+c)^2}=\frac{4}{\sqrt{t+4}}-\frac{9}{2t}=f(t)$ với $t=\frac{(a+b+c)^2}{3}$  $(t>0)$

$f'(t)=\frac{-2}{\sqrt{(t+4)^3}}+\frac{9}{2t^2}$   ;   $f'(t)=0\Leftrightarrow 16t^4=81(t+4)^3$

Nhận xét :

  • Với $0<t<12$ thì $16t^4<16.12t^3=3.(4t)^3<3.(3t+12)^3=81(t+4)^3$ tức là $f'(t)>0$ trong $(0;12)$
  • Với $t>12$ thì $16t^4>16.12t^3=3.(4t)^3>3.(3t+12)^3=81(t+4)^3$ tức là $f'(t)<0$ trong $(12;+\infty)$
  • Với $t=12$ thì $f'(t)=0$ và $f(t)=\frac{5}{8}$

Suy ra : $\max_{t>0} f(t)=\frac{5}{8}$ tại $t=12$

Vậy $\max_{a,b,c>0}P=\frac{5}{8}$ khi và chỉ khi $a=b=c=2$.




#434034 [TSĐH 2013] Đề thi môn toán khối B

Đã gửi bởi Kool LL on 09-07-2013 - 16:49 trong Thi TS ĐH

Câu 6. Dùng AM-GM đánh giá được :

$P\le \frac{4}{\sqrt{t+4}}-\frac{9}{2t}=f(t)$ với $t=\frac{(a+b+c)^2}{3}$  $(t>0)$

 

Cách 2 : Áp dung BĐT quen thuộc $\frac{4}{\sqrt{a_1+a_2+a_3+a_4}}\le\sqrt{\frac{1}{a_1}+\frac{1}{a_2}+\frac{1}{a_3}+\frac{1}{a_4}}$ ( Dấu $"="$ xảy ra với $a_1=a_2=a_3=a_4$)

Ta có : $\frac{4}{\sqrt{t+4}}=\frac{4}{\sqrt{\frac{t}{3}+\frac{t}{3}+\frac{t}{3}+4}}\le\sqrt{\frac{9}{t}+\frac{1}{4}}$ Dấu $"="$ xảy ra với $t=12$

Suy ra $P\le\sqrt{\frac{9}{t}+\frac{1}{4}}-\frac{9}{2t}$. Đặt $z=\sqrt{\frac{9}{t}+\frac{1}{4}}$  $(z>\frac{1}{4})$  thì

$P\le z-\frac{1}{2}\left( z^2-\frac{1}{4}\right) =\frac{1+8z-4z^2}{8}=\frac{5-4(1-z)^2}{8}\le\frac{5}{8}$

Dấu $"="$ xảy ra với $z=1\Leftrightarrow t=12\Leftrightarrow a=b=c=2$




#434049 [TSĐH 2013] Đề thi môn toán khối B

Đã gửi bởi Kool LL on 09-07-2013 - 18:18 trong Thi TS ĐH

Cách 2 : Áp dung BĐT quen thuộc $\frac{4}{\sqrt{a_1+a_2+a_3+a_4}}\le\sqrt{\frac{1}{a_1}+\frac{1}{a_2}+\frac{1}{a_3}+\frac{1}{a_4}}$ ( Dấu $"="$ xảy ra với $a_1=a_2=a_3=a_4$)

Ta có : $\frac{4}{\sqrt{t+4}}=\frac{4}{\sqrt{\frac{t}{3}+\frac{t}{3}+\frac{t}{3}+4}}\le\sqrt{\frac{9}{t}+\frac{1}{4}}$ Dấu $"="$ xảy ra với $t=12$

Suy ra $P\le\sqrt{\frac{9}{t}+\frac{1}{4}}-\frac{9}{2t}$. Đặt $z=\sqrt{\frac{9}{t}+\frac{1}{4}}$  $(z>\frac{1}{4})$  thì

$P\le z-\frac{1}{2}\left( z^2-\frac{1}{4}\right) =\frac{1+8z-4z^2}{8}=\frac{5-4(1-z)^2}{8}\le\frac{5}{8}$

Dấu $"="$ xảy ra với $z=1\Leftrightarrow t=12\Leftrightarrow a=b=c=2$

Hoặc có thể tiếp tục dùng Côsi :

$\sqrt{\frac{9}{t}+\frac{1}{4}}=1.\sqrt{\frac{9}{t}+\frac{1}{4}}\le\frac{1+\left(\frac{9}{t}+\frac{1}{4}\right)}{2}=\frac{9}{2t}+\frac{5}{8}$

$\Rightarrow P\le \frac{5}{8}$




#440617 [TSĐH 2013] Đề thi môn toán khối B

Đã gửi bởi Kool LL on 05-08-2013 - 15:57 trong Thi TS ĐH

Mọi người cho minh hỏi câu 1b mình lam như thế này liệu có được điểm nào không

     Ta có y'=$6x^2+6(m+1)x+6m$

 

Để h/s có cực đai,cực tiêu thì pt y'=0 có 2 nghiệm pb 

$\Leftrightarrow \Delta >0\Leftrightarrow (m-1)^2>0\Leftrightarrow m>1$(do tâm lí phòng thi),

Với m>1,lại có $y=\frac{1}{3}x.y'-\frac{1}{6}(m+1)y'-(m-1)^2x+....$

Suy ra đt AB có pt y=$-(m-1)^2x+....$

Do  AB vuuông góc với d  $\Rightarrow (m-1)^2=1\Leftrightarrow m=0;m=2$$\Rightarrow (m-1)^2=1\Leftrightarrow m=0;m=2$

Với chỗ sai đó thì bạn được câu này từ 0.5 -> 0.75 đ. Khi chấm thi thì tui cho những ai sai chỗ đó 0.75 đ (với bài làm hoàn chỉnh y như bài giải của bạn).




#441005 Tìm x,n nguyên dương sao cho $x^3+2x+1=2^n$

Đã gửi bởi Kool LL on 07-08-2013 - 13:05 trong Số học

Tìm x,n nguyên dương sao cho $x^3+2x+1=2^n$  (*)

p/s: Bài này mình giải đc theo Legendre rồi, ai có cách khác chỉ mình với!

Vì $x,n$ nguyên dương nên từ (*) $\Rightarrow x$ lẻ và $n\ge2$

(*) $\Leftrightarrow (x+1)(x^2-x+3)=2.(2^{n-1}+1)$ (**)

Vì x lẻ nên $(x^2-x+3)$ cũng lẻ và $(x+1)$ chẵn. Mà $(2^{n-1}+1)$ lẻ do đó từ (**) suy ra $x+1=2$ và ta được $x=1$.

Thay $x=1$ vào (*) suy ra $n=2$.

Vậy ta có (x,n)=(1,2) là nghiệm nguyên duy nhất của (*).




#441075 Tìm x,n nguyên dương sao cho $x^3+2x+1=2^n$

Đã gửi bởi Kool LL on 07-08-2013 - 18:07 trong Số học

Chưa hẳn vậy.nếu x+1 =4hoặc 6,8  vẫn đảm bảo là số chẵn mà

Vì VT có ước số chẵn là $x+1$, VP chỉ có ước số chẵn duy nhất là 2. Muốn VT=VP thì phải có $x+1=2$.




#441662 đề thi học sinh giỏi thành phố hà nội câu 1 số học

Đã gửi bởi Kool LL on 10-08-2013 - 04:36 trong Số học

Câu 1. (4 điểm)
Giả sử n là số tự nhiên khác 0 sao cho 2n và 5n bắt đầu cùng bằng chữ số a. Hãy tìm chữ số a.

 

Bổ đề : Số chữ số của $x^n$ là $S(x^n)=[n.\lg x]+1$. (kí hiệu [A] nghĩa là phần nguyên của số thực A)

 

CM bổ đề : Đặt $S(x^n)=k$ , do $S(10^{k-1})=k$ và $S(10^k)=k+1$ nên : $10^{k-1}\le x^n<10^k$ .

Lấy $\log_{10}$ cả 3 vế, ta được : $k-1\le n.\lg x<k$

$\Rightarrow [n.\lg x]=k-1 \Rightarrow S(x^n)=k=[n.\lg x] +1$. (CM xong bổ đề).

 

Trở lại bài toán, do $2^n$ và $5^n$ đều bắt đầu bằng chữ số $a$ nên ta có thể viết như sau :

$2^n=a.10^{k_1}+r_1$ với $0<r_1<10^{k_1}$ và $k_1=[n.\lg{2}]$

$5^n=a.10^{k_2}+r_2$ với $0<r_2<10^{k_2}$ và $k_2=[n.\lg{5}]$

$\Rightarrow 10^n=2^n.5^n=a^2.10^{k_1+k_2}+a.(r_1.10^{k_2}+r_2.10^{k_1})+r_1.r_2$

$\Rightarrow a^2.10^{k_1+k_2}<10^n<a^2.10^{k_1+k_2}+a.2.10^{k_1+k_2}+10^{k_1+k_2}$

$\Rightarrow a^2.10^{k_1+k_2}<10^n<(a+1)^2.10^{k_1+k_2}$

$\Rightarrow a<\sqrt{10^{n-k_1-k_2}}<a+1$

$\Rightarrow a=[\sqrt{10^{n-k_1-k_2}}]$

Mà $n-2=(n\lg 2-1)+(n\lg 5-1)<k_1+k_2<n\lg 2+n\lg 5=n \Rightarrow k_1+k_2=n-1$

Vậy $a=[\sqrt{10}]=3$.




#442478 $f(a,b,c)= \dfrac{a}{b^k+c^k} +\dfrac...

Đã gửi bởi Kool LL on 13-08-2013 - 15:21 trong Bất đẳng thức - Cực trị

ở bài này mình chỉ tìm được giá trị lớn nhất chứ không tìm được giá trị nhỏ nhất


áp dụng bdt cô si ta được
$b^{k}+a^{k}\geq 2\sqrt{a^{k}b^{k}}$
$b^{k}+c^{k}\geq 2\sqrt{c^{k}b^{k}}$
$c^{k}+a^{k}\geq 2\sqrt{a^{k}c^{k}}$
Suy ra: $f(a,b,c)\leq \frac{a}{2\sqrt{b^{k}c^{k}}}+\frac{b}{2\sqrt{a^{k}c^{k}}}+\frac{c}{2\sqrt{b^{k}a^{k}}}= \frac{a\sqrt{a^{k}}+b\sqrt{b^{k}}+c\sqrt{c^k}}{2\sqrt{a^kb^kc^k}}$ (1)
Áp dụng bdt bu-nhi-a cốp xki ta có : $a\sqrt{a^k}+b\sqrt{b^k}+c\sqrt{c^k}\leq \sqrt{(a^{2}+b^2+c^2)(a^k+b^k+c^k)}= \sqrt{a^{k}+b^{k}+c^{k}}$ (2)
Dấu bằng xảy ra khi và chỉ khi $a^{k}= b^{k}=c^{k}\Leftrightarrow a^2=b^2=c^2=\frac{1}{3}$ (3)

Từ(1) (2)và (3) suy ra f(a,b,c) max

 

Khi có $f(x)\le g(x)$, dấu $"="$ xảy ra khi và chỉ khi $x=x_0$ mà $g(x)$ không phải hằng số thì không thể kết luận $\max f(x)=g(x_0)$ tại $x=x_0$ được.

 

Không có GTLN của $f(a,b,c)$. Chẳng hạn cho $b\to 0^+, c\to 0^+$ thì $a\to 1^-$ và ta có $f(a,b,c)\to +\infty$




#442516 CMR $\exists n \in N$ để $n^2+1 \vdots p$

Đã gửi bởi Kool LL on 13-08-2013 - 16:56 trong Số học

Chứng minh rằng với mọi $p$ là số nguyên tố thì đều tồn tại $n \in N$ thỏa mãn $p <\frac{n}{2}$ mà $n^2+1 \vdots p$

Đề bài này có vấn đề rồi. Phản ví dụ đây : xét p=3 thì không tồn tại $n\in \mathbb{N}$ thoả mãn $\frac{n}{2}>3$ mà $n^2+1$ $\vdots$ $3$.

Thật vậy, $\forall n\in\mathbb{N}$ ta có $n\equiv r\pmod{3}$ với $r\in\{0,1,2\}$. $\Rightarrow n^2+1\equiv r^2+1\equiv s\pmod{3}$ với $s\in\{1,2\}\Rightarrow n^2+1$ không $\vdots 3$.(đpcm)

 

1) Nếu $p=2$ thì chọn $n=pk+1,(k>2)$. Ta có : $n>2p$ và $n^2+1$ $\vdots p$.

2) Nếu $p>2$ thì $p$ là số nguyên tố lẻ. Ta có : $\exists\in\mathbb{N}, n^2+1$ $\vdots$ $p\Leftrightarrow p$ có dạng $4k+1$.




#444813 $f(a,b,c)= \dfrac{a}{b^k+c^k} +\dfrac...

Đã gửi bởi Kool LL on 22-08-2013 - 21:27 trong Bất đẳng thức - Cực trị

bạn thử xem lại đi. khi mjnh áp dụng bdt cô si cho 2 số ko âm. mk a,b,c ko phải là hằng số mà k lại thay đổi nên mình có thể tìm a,b,c để f(a,b,c) max

 

Đề bài nói cho trước số $k>0$ có nghĩa là cho trước hằng số k rồi. Chỉ có $a,b,c>0$ và thay đổi sao cho thoả $a^2+b^2+c^2=1$ thôi.

 

Phản chứng : Giả sử có GTLN của $f(a,b,c)$.

Tức là $\exists$ hằng số $M>0$ sao cho $f(a,b,c)\le M$, $\forall a,b,c>0$ thoả $a^2+b^2+c^2=1$          .(1)

       và $\exists a_0,b_0,c_0>0$ sao cho $a_0^2+b_0^2+c_0^2=1$ thoả $f(a_0,b_0,c_0)=M$

Chọn $0<b=c=\epsilon<\frac{1}{\sqrt{2}},0<a=\sqrt{1-2\epsilon^2}$ thì $a^2+b^2+c^2=1$ và $f(a,b,c)=\frac{\sqrt{1-2\epsilon^2}}{2\epsilon^k}}+\frac{2\epsilon}{\epsilon ^k+\sqrt{1-2\epsilon^2}$ $\to+\infty$ khi $\epsilon\to0^+$.   (2)

Từ (1) suy ra, theo nguyên lý kẹp trong giới hạn, ta có : $\lim_{\epsilon\to 0^+}f(a,b,c)\le M$.   (3)

Từ (2)(3) $\Rightarrow +\infty\le M$ (Vô lý vì $M$ là hằng số)

 

 

theo cách bạn đang làm thì là tìm min của cả bdt chứ ko pải là tìm min kủa x nk. pạn sai rồi kìa. tìm một cái ví du khác đúng hơn đi ha. ùi khi đó mình sẽ nhận sai

 

@suchica cần phải học lại ĐỊNH NGHĨA GTLN của một hàm f(x) đi nha.

$f(x)$ xác định trên $D$. Khi đó :

$f(x)$ đạt GTLN bằng $M$ tại $x=x_0\Leftrightarrow \left\{\begin{matrix} \exists M(const) : f(x)\le M ,\forall x\in D; \color{Red}{(1)}\\ \exists x_0\in D : f(x_0)=M. \cổl{Red}{(2)} \end{matrix}\right.$

Ở đây $x$ cũng có thể hiểu là nhiều biến, chẳng hạn bộ số $x$ là $(x_1,x_2,...,x_n)$, và tương ứng có bộ số $x_0$ là $(x_{0_1},x_{0_2},...,x_{0_n})$.

Nếu không có M là hằng số ở đk(1) và không tìm được $x_0$ ở đk(2) thì không thể kết luận $f(x)$ đạt GTLN.

Ví dụ :

Ta dễ dàng chứng minh được $f(x)=1-x^4\le 2-2x^2=g(x),\forall x$. Dấu $"="$ xảy ra khi và chỉ khi $x=\pm1$.

Đến đây ta không thể khẳng định $f(x)$ đạt GTLN tại $x_0=\pm1$. Vì $f(x_0=\pm1)=0<f(0)=1$.

Ngoài ra, $g(x)\le2,\forall x$ và $g(x_0=0)=2$ nên có thể khẳng định $g(x)$ đạt GTLN bằng $2$ tại $x_0=0$.

Nhưng $f(x)\le g(x)\le2$ thì không thể khẳng định $f(x)$ đạt GTLN bằng 2 vì không $\exists x_0$ để $f(x_0)=2$.

Bởi vì $f(x)=1-x^4\le1,\forall x$ và $f(x_0=1)=1$ nên có thể khẳng định $f(x)$ đạt GTLN bằng $1$ tại $x_0=0$.

Như vậy thì ta chỉ có $f(x)<2$ chứ không có $x_0$ để $f(x_0)=2$ được.




#444880 Topic về số học, các bài toán về số học.

Đã gửi bởi Kool LL on 23-08-2013 - 15:10 trong Số học

Bài 17: Cho n là một số nguyên dương thỏa mãn 2n + 1; 3n + 1 đều là số chính phương. Hỏi  5n + 3 là số nguyên tố hay hợp số? Tìm số n nhỏ nhất thỏa tất cả điều kiện đó.

(Cải biên lại từ đề đề nghị thi Olimpic 30/4 lớp 10 năm 2009 của Trường THPT chuyên KonTum - Sở GD - ĐT KonTum)

 

Giả sử $2n+1=a^2, 3n+1=b^2$. (1) $\Rightarrow 3a^2-2b^2=1$ và $b^2-a^2=n$. (2)

Do $n\in\mathbb{Z}^+$ nên $a\ge 2, b\ge 3, b>a$.

Ta có : $5n+3=4a^2-b^2=(2a-b)(2a+b)$. (3)

Nếu $2a-b=1$ thì $b=2a-1$, thay vào (2) được $n=3a^2-4a+1$, thay tiếp vào (1) được $5a^2-8a+3=0$ (VN do $a\ge 2$).

Vậy $2a+b>2a-b>1$. Từ (3)$\Rightarrow 5n+3$ có 2 ước lớn hơn 1 nên là hợp số.

 

$(a^2,b^2)=(2n+1,3n+1)=(2n+1,n)=(n+1,n)=1\Rightarrow (a,b)=1$.

Như vậy việc tìm $n$ thoả đề bài chính là tìm bộ nghiệm nguyên (a,b) nguyên tố cùng nhau của PT $3a^2-2b^2=1$.

(Xem tiếp bài giải ở bên dưới #99).




#444882 Topic về số học, các bài toán về số học.

Đã gửi bởi Kool LL on 23-08-2013 - 15:21 trong Số học

Bài 17 : (tiếp theo ý 2)

Như vậy việc tìm $n=b^2-a^2$  chính là tìm bộ nghiệm nguyên $(a,b)$ của PT $3a^2-2b^2=1$.

 

PT $Aa^2-Bb^2=1$ (1) là PT Pell dạng tổng quát, ta sẽ chuyển về PT Pell loại 1 như sau :

Đặt $x=Aa^2+Bb^2, y=2ab,d=AB$ thì $x^2-dy^2=(Aa^2+Bb^2)^2-AB(2ab)^2=(Aa^2-Bb^2)^2=1^2=1$

 

$x^2-dy^2=1$ (2) là PT Pell loại 1, nếu d không chính phương thì với $(x_1,y_1)$ là nghiệm cơ sở (nhỏ nhất), ta có thể tìm $(x_n,y_n)$ là tất cả các nghiệm của (2), bằng một trong các công thức sau :

a) Công thức khai triển và đồng nhất :

$x_n+y_n\sqrt{d}=(x_1+y_1\sqrt{d})^n$

b) Công thức truy hồi :

$x_{n+1}=x_1x_n+dy_1y_n$

$y_{n+1}=x_1y_n+y_1x_n$

c) Công thức tong quát :

$x_n=\frac{(x_1+y_1\sqrt{d})^n+(x_1-y_1\sqrt{d})^n}{2}; y_n=\frac{(x_1+y_1\sqrt{d})^n-(x_1-y_1\sqrt{d})^n}{2\sqrt{d}}$

 

 

 

Từ đây, ta dễ dàng chứng minh được các kết quả sau :

 

1/. Nếu $(a_1,b_1)$ là nghiệm cơ sở (nhỏ nhất) của PT (1) thì $(a_1,b_1)$ là nghiệm của HPT

$\left\{\begin{matrix}Aa^2+Bb^2=x_1 \\ 2ab=y_1 \end{matrix}\right.$ ,    (với $(x_1,y_1)$ là nghiệm cơ sở của PT (2) ).

 

 2/. Nếu $(a_1,b_1)$ là nghiệm cơ sở của PT(1) thì có thể tìm $(a_n,b_n)$ là tất cả các nghiệm của PT (1) bằng một trong các công thức sau :

a) Công thức khai triển và đồng nhất:

$a_n\sqrt{A}+b_n\sqrt{B}=(a_1\sqrt{A}+b_1\sqrt{B})^{2n-1}$

b) Công thức truy hồi:

$a_{n+1}=(Aa_1^2+Bb_1^2)a_n+(2a_1b_1)Bb_n$

$b_{n+1}=(Aa_1^2+Bb_1^2)b_n+(2a_1b_1)Aa_n$

c) Công thức tổng quát:

$a_n=\frac{(a_1\sqrt{A}+b_1\sqrt{B})^{2n-1}+(a_1\sqrt{A}-b_1\sqrt{B})^{2n-1}}{2\sqrt{A}}; b_n=\frac{(a_1\sqrt{A}+b_1\sqrt{B})^{2n-1}-(a_1\sqrt{A}-b_1\sqrt{B})^{2n-1}}{2\sqrt{B}}$

 

Áp dung vào bài toán, ta tìm được tất cả các bộ nghiệm của PT $3a^2-2b^2=1$ là : (1,1); (9,11); (89,109); ...

Tương ứng với n=0 (loại); n=40 (nhận); n=3960; ...

Vậy $n=40$ là số nhỏ nhất thoả $2n+1$ và $3n+1$ là các số chính phương.




#444993 Topic về số học, các bài toán về số học.

Đã gửi bởi Kool LL on 23-08-2013 - 21:51 trong Số học

Bài 21: Giải phương trình nghiệm nguyên : $xyz=x^{2}-2z+2$. (1)

 

(1)$\Leftrightarrow z(xy+2)=x^2+2 \Rightarrow z\ne0$.

* Nếu $x=0$ thì $z=2$ và $y\in\mathbb{Z}$ bất kỳ. Nghiệm $(0,t,1)$ với $t\in\mathbb{Z}$.

* Nếu $y=0$ thì $x^2-2z+2=0\Leftrightarrow x^2=2(z-1)$. Nghiệm $(2t,0,2t^2+1)$ với $t\in\mathbb{Z}$.

* Xét $xyz\ne0$:

Nhận xét : nếu $(x,y,z)$ là nghiệm thì $(-x-y,z)$ cũng là nghiệm. Nên không mất tính tổng quát, chỉ cần xét $x>0$. Khi đó nếu $y>0$ thì $z>0$ và nếu $z<0$ thì $y<0$. Do đó ta chỉ có 3 TH sau :

  • TH1: $y<0<z$ thì $0>xy>-2\Rightarrow xy=-1\Rightarrow$ Nghiệm là $(1,-1,3)$ và $(-1,1,3)$.
  • TH2: $y,z>0$ thì $z=\frac{x^2+2}{xy+2}\ge1\Rightarrow x\ge y; x-yz=x-\frac{x^2y+2y}{xy+2}=\frac{2(x-y)}{xy+2}\ge0$
    • nếu $z=1$ thì $x=y$. Suy ra nghiệm là $(t,t,1)$, với $t\in\mathbb{Z}$.
    • nếu $z>1$ thì $x>yz>y$ và $2(x-y)\ge xy+2\Rightarrow (x+2)(2-y)\ge6\Rightarrow 0<y<2\Rightarrow y=1$

$\Rightarrow z=\frac{x^2+2}{x+2}=x-2+\frac{6}{x+2}\Rightarrow x+2=6\Rightarrow x=4$.

Vậy nghiệm là $(4,1,3)$ và $(-4,-1,3)$.

  • TH3: $y,z<0$. Đặt $a=x,b=-y,c=-z$ thì $a,b,c>0$ thoả $abc=a^2+2c+2$ $\Rightarrow ab>2$ và $bc>a$ $\Rightarrow c=\frac{a^2+2}{ab-2}\Rightarrow bc-a=b.\frac{a^2+2}{ab-2}-a=\frac{2(a+b)}{ab-2}$ $\Rightarrow 2(a+b)\ge ab-2 \Rightarrow (a-2)(b-2)\le6$
    • nếu $b=1$ thì $c=\frac{a^2+2}{a-2}=a+2+\frac{6}{a-2}\Rightarrow a-2\in\{1;2;3;6\}\Rightarrow a\in\{3,4,5,8\}$. Nghiệm $(a,b,c)$ là $(3,1,11); (4,1,9); (5,1,9); (8,1,11)$. Nghiệm $(x,y,z)$ là $(3,-1,-11); (4,-1,-9); (5,-1,-9); (8,-1,-11)$ và $(-3,1,-11); (-4,1,-9); (-5,1,-9); (-8,1,-11)$.
    • nếu $b=2$ thì $2c-a=\frac{2(a+2)}{2a-2}=1+\frac{3}{a-1}\Rightarrow a-1\in\{1,3\}\Rightarrow a\in\{2,4\}$. Nghiệm $(a,b,c)$ là $(2,2,3); (4,2,3)$. Nghiệm $(x,y,z)$ là $(2,-2,-3); (4,-2,-3)$ và $(-2,2,-3); (-4,2,-3)$.
    • nếu $a=1$ thì $c=\frac{3}{b-2}\Rightarrow b-2\in\{1,3\}\Rightarrow b\in\{3,5\}$. Nghiệm $(a,b,c)$ là $(1,3,3); (1,5,1)$. Nghiệm $(x,y,z)$ là $(1,-3,-3); (1,-5,-1)$ và $(-1,3,-3); (-1,5,-1)$.
    • nếu $a=2$ thì $c=\frac{3}{b-1}\Rightarrow b-1\in\{1,3\}\Rightarrow b\in\{2,4\}$. Nghiệm $(a,b,c)$ là $(2,2,3); (2,4,1)$. Nghiệm $(x,y,z)$ là $(2,-2,-3); (2,-4,-1)$ và $(-2,2,-3); (-2,4,-1)$.
    • nếu $a,b>2$ thì $ab-2\ge7$ và $(a-2)(b-2)=k, 0<k\le6$. $\Rightarrow 2(a+b)=ab+4-k\Rightarrow bc-a=\frac{ab+4-k}{ab-2}=1+\frac{6-k}{ab-2}$. 
      • nếu $0<k<6\Rightarrow (6-k)\vdots(ab-2)\Rightarrow 6-k\ge ab-2\ge7\Rightarrow k\le-1$. (Vô lý !)
      • nếu $k=6\Rightarrow bc-a=1\Rightarrow bc=a+1$ $\Rightarrow a^2+a=abc=a^2+2c+2\Rightarrow a=2c+2\Rightarrow bc=2c+3 \Rightarrow c=\frac{3}{b-2}$ $\Rightarrow b-2\in\{1,3\} \Rightarrow b\in\{3,5\}$. Nghiệm $(a,b,c)$ là $(8,3,3); (4,5,1)$. Nghiệm $(x,y,z)$ là $(8,-3,-3); (4,-5,-1)$ và $(-8,3,-3); (-4,5,-1)$.

 

Kết luận : Nghiệm của PT(1) là $(0,t,1); (2t,0,2t^2+1); (t,t,1)$ với $t\in\mathbb{Z}$;

$(1,-1,3)$ và $(-1,1,3)$;

$(4,1,3)$ và $(-4,-1,3)$;

$(3,-1,-11), (4,-1,-9), (5,-1,-9), (8,-1,-11)$ và $(-3,1,-11), (-4,1,-9), (-5,1,-9), (-8,1,-11)$;

$(2,-2,-3), (4,-2,-3)$ và $(-2,2,-3), (-4,2,-3)$;

$(1,-3,-3), (1,-5,-1)$ và $(-1,3,-3), (-1,5,-1)$;

$(2,-2,-3), (2,-4,-1)$ và $(-2,2,-3), (-2,4,-1)$;

$(8,-3,-3), (4,-5,-1)$ và $(-8,3,-3), (-4,5,-1)$.




#445029 Topic về số học, các bài toán về số học.

Đã gửi bởi Kool LL on 24-08-2013 - 05:28 trong Số học

Bài 27: Cho 2 bộ ba số nguyên dương a, b, c và d, e, f sao cho $(a,b,c)=1;(d,e,f)=1$ (1) và thoả mãn đẳng thức $\frac{1}{a}+\frac{1}{b}=\frac{1}{c}$; $\frac{1}{d}+\frac{1}{e}=\frac{1}{f}$ (2). Chứng minh rằng 2(a + b + d + e) là tổng bình phương của hai số tự nhiên.  

 

Nếu $a=b$ thì $\overset{(2)}{\Rightarrow} a=b=2c$ $\overset{(1)}{\Rightarrow} 1=(a,b,c)=(2c,2c,c)=c$ $\Rightarrow a=b=2\Rightarrow a+b=4=2^2=(c+1)^2$.

Giả sử $a>b$ thì $a>2c>b>c\Rightarrow a-c>c>b-c\ge1$.

Gọi $d=(b,c)$ thì $b=db_1; c=dc_1; (b_1,c_1)=1$.

$(2)\Rightarrow ab=c(a+b)\Rightarrow ab_1=c_1(a+db_1)\Rightarrow a\vdots b_1, a\vdots c_1\Rightarrow a\vdots b_1c_1$ $\Rightarrow a=k.b_1c_1\Rightarrow k(b_1-c_1)=d\Rightarrow k|d|b,c$ và $k|a\Rightarrow k=1$

$\Rightarrow a=b_1c_1; b_1-c_1=d\Rightarrow b-c=d^2\Rightarrow a+b=b_1c_1+c+d^2=b_1c_1+(b_1-c_1)c_1+$$(b_1-c_1)^2=b_1^2$

Vậy ta luôn có $a+b$ là số chính phương $m^2$. Tương tự với $d+e=n^2$.

Suy ra $2(a+b+d+e)=2(m^2+n^2)=(m+n)^2+(m-n)^2$ là tổng bình phương của hai số tự nhiên. (đpcm)




#445031 Topic về số học, các bài toán về số học.

Đã gửi bởi Kool LL on 24-08-2013 - 06:08 trong Số học

Giải bài 27 (đã hết hạn bỏ bài  :icon6: ): Trước hết ta sẽ chứng minh rằng a + b và c + d đều là số chính phương.

Vì (a, b, c) = 1 tức là a, b, c ko có nhân tử chung thế nên ta hoàn toàn có thể giả sử $b\vdots c,(a,c)=1$. ...

 

Căn cứ nào mà lại có thể giả sử như thế !!? Bạn chứng minh được điều đó không ??




#445032 Topic về số học, các bài toán về số học.

Đã gửi bởi Kool LL on 24-08-2013 - 06:17 trong Số học

Bài này mình chỉ giải được trên tập nghiệm tự nhiên nên trình bày cách giải ra để mọi người cùng góp ý, nếu ai giải được trên tập nghiệm nguyên thì bổ sung cho bài của mình nhé !

 

$PT\Leftrightarrow z(xy+2)=x^{2}+2\Rightarrow z=\frac{x^{2}+2}{xy+2}\in N$

$\Rightarrow x+\frac{2(y-x)}{xy+2}\in Z\Rightarrow 2(y-x)=k(xy+2)$ với $k$ tự nhiên

 

  • Xét $x=1$ ta có $yz=3-2z\Leftrightarrow z(y+2)=3\Rightarrow y=z=1$
  • Xét $x\geq 2$ :

Nếu $k = 0$ thì $x^{2}+2=0$ (loại)

Nếu $k\geq 1$ thì $2(y-x)\geq xy+2\Leftrightarrow (x-2)(y+2)+6\leq 0$

Nhưng $x\geq 2,y\geq 0\Rightarrow (x-2)(y+2)+6>0$

Phương trình vô nghiệm

Vậy : Phương trình có nghiệm tự nhiên duy nhất $(1 ; 1; 1)$

 

$k\in\mathbb{Z}$ mà, sao lại cho $\in\mathbb{N}$ luôn vậy !!!

 

$k=0$ thì $x=y$ và PT có VSN dạng $(t,t,1)$ mà !!!




#445034 Topic về số học, các bài toán về số học.

Đã gửi bởi Kool LL on 24-08-2013 - 06:54 trong Số học

Bài 29

Cho $p_{n}$ là số nguyên tố thứ $n$. Chứng minh rằng:

a.$p_{n}>2n$ với mọi $n>4$

b.$p_{n}>3n$ với mọi $n>11$

 

a) Ta CM bang pp qui nạp theo $n>4$.

  • $n=5$: $p_5=11>2.5=10$ (đúng).
  • G/s bài toán đúng đến $n=k, (k>4)$, tức là ta có $p_n>2n \forall n=\overline{5..k}$.
  • Xét n=k+1: Do $p_{k+1} > p_k$ là hai số nguyên tố lẻ liên tiếp nên $p_{k+1}\ge p_k+2$.

Dùng giả thiết qui nạp, ta có $p_k>2k$. Suy ra $p_{k+1}>2k+2=2(k+1)$

Như vậy bài toán cũng đúng với $n=k+1$.

Theo nguyên lý quy nạp suy ra bài toán đúng $\forall n>4$. (đpcm)

 

b)  Ta CM bang pp qui nap theo $n>11$.

  • $n=12$: $p_{12}=37>3.12=36$ (đúng).
  • G/s bài toán đúng đến $n=k, (k>11)$, tức là ta có $p_n>3n \forall n=\overline{12..k}$.
  • Xét n=k+1: Do $p_{k+1} > p_k$ là hai số nguyên tố lẻ lien tiếp nên $p_{k+1}\ge p_k+2$.

Dùng giả thiết qui nạp, ta có $p_k>3n$, tức là $p_k\ge 3n+1$. Suy ra $p_{k+1}\ge 3k+1+2=3(k+1)$.

Dấu $"="$ không thể xảy ra vì khi đó $p_{k+1}\vdots3$ (mâu thuẫn với $p_{k+1}$ là số nguyên tố).

Vậy $p_{k+1}>3(k+1)$. Bài toán cũng đúng với $n=k+1$.

Theo nguyên lý qui nạp suy ra bài toán đúng $\forall n>11$. (đpcm)




#445112 Topic về số học, các bài toán về số học.

Đã gửi bởi Kool LL on 24-08-2013 - 16:34 trong Số học

Bài 31:

 Cho số tự nhiên y .Chứng minh tồn tại vô số nguyên tố p sao cho d08057d8d86e79b7b0888b1fc06d3ef6_4.0pt.p và 72e8d7b77259e1ece54a452e313d04d0_4.0pt.p.

 

Viết đề kiểu này khó hiểu quá! Phát biểu lại ý của đề bài cho dễ hiểu hơn đây :

"Cho trước số tự nhiên $y$. CMR : Có vô số số tự nhiên $n$ để số $(2^ny+1)$ có ước nguyên tố dạng $4k+3$."

 




#445151 Chứng minh rằng $P(x)$ bất khả quy trên $\mathbb{Z...

Đã gửi bởi Kool LL on 24-08-2013 - 19:10 trong Đa thức


Cho đa thức $P(x)$ hệ số nguyên bậc $n$. Biết $P(x)$ nhận giá trị $1$ hoặc $-1$ tại nhiều hơn $2\left [ \frac{n}{2} \right ]$ điểm nguyên phân biệt. Chứng minh rằng $P(x)$ bất khả quy trên $\mathbb{Z}\left [ x \right ]$

Ta CM bằng phản chứng.

Giả sử $P(x)$ khả qui trên $\mathbb{Z}[x]$. Tức là $\exists$ các đa thức $A(x),B(x)$ hệ số nguyên bậc tương ứng $a\ge b$.

  • nếu $n$ chẵn thì $n-1\ge a\ge \left[\frac{n}{2}\right] \ge b\ge1$
  • nếu $n$ lẻ thì $n-1\ge a\ge \left[\frac{n}{2}\right]+1>\frac{n}{2}>\left[\frac{n}{2}\right] \ge b\ge1$

Do đó ta luôn có $\forall n: \left[\frac{n}{2}\right] \ge b\ge1\Rightarrow 2\left[\frac{n}{2}\right] \ge 2b\ge2$

$\Rightarrow B^2(x)$ là đa thức hệ số nguyên có bậc là $2b$ và không quá $2\left [ \frac{n}{2} \right ]$. (1)

Mặt khác, $P^2(x)=A^2(x).B^2(x)=1\Leftrightarrow A^2(x)=B^2(x)=1$ tại nhiều hơn $2\left [ \frac{n}{2} \right ]$ điểm nguyên phân biệt. (Do $A^2(x),B^2(x)$ chỉ nhận các giá trị nguyên không âm).

Suy ra PT $B^2(x)=1$ có nhiêu hơn $2\left [ \frac{n}{2} \right ]$ nghiệm nguyên phân biệt. (2)

Ta thấy $(1)$ và $(2)$ mâu thuẫn nhau !!

Vậy $P(x)$ bất khả qui trên $\mathbb{Z}[x]$. (đpcm)




#445160 $(\sqrt{3}+1)^{log_{2}x}+x.(\sqr...

Đã gửi bởi Kool LL on 24-08-2013 - 19:40 trong Phương trình - hệ phương trình - bất phương trình



Giải phương trình:

$(\sqrt{3}+1)^{log_{2}x}+x.(\sqrt{3}-1)^{log_{2}x}=1+x^{2}$. (1)

Điều kiện : $x>0$

Đặt $y=(\sqrt{3}+1)^{\log_2{x}} \leftrightarrow \ln{x}=\frac{\ln{2}}{\ln{(\sqrt{3}+1)}}.\ln{y}$

và $y.(\sqrt{3}-1)^{\log_2{x}}=2^{\log_2{x}}=x$

PT $(1)$ trở thành : $y+x.\frac{x}{y}=1+x^2$

$\Leftrightarrow (y-1)(y-x^2)=0$

$\Leftrightarrow \left[\begin{matrix}y=1 \\ y=x^2\end{matrix}\right.$

$\Leftrightarrow \left[\begin{matrix}\ln{x}=0 \\ 2\ln{x}=\ln{y}=\frac{\ln{(\sqrt{3}+1)}}{\ln{2}}.\ln{x}\end{matrix}\right.$

$\Leftrightarrow \ln{x}=0$

$\Leftrightarrow x=1$. (nhận)

Vậy PT chỉ có một nghiệm $x=1$.




#445245 $I=\int_{-1}^{1}\frac{dx}{1...

Đã gửi bởi Kool LL on 25-08-2013 - 03:50 trong Tích phân - Nguyên hàm

Tính tích phân:

$I=\int_{-1}^{1}\frac{dx}{1+x+\sqrt{1+x^{2}}}$

 

$I=\int_{-1}^{1}\frac{1+x-\sqrt{1+x^2}}{2x}dx$ $=\frac{1}{2}.\left[ \int_{-1}^{1}\frac{dx}{x}+\int_{-1}^{1}dx-\int_{-1}^{1}\frac{\sqrt{1+x^2}}{x}dx\right]$ $=\frac{1}{2}.(I_1+I_2-I_3)$

 

Định lý :

a) Nếu $f(x)$ là hàm số lẻ, tức là $f(-x)=-f(x)$, thì $\int_{-a}^{a}f(x)dx=0$.

b) Nếu $f(x) $ là hàm số chẵn, tức là $f(-x)=f(x)$, thì $\int_{-a}^{a}f(x)dx=2.\int_{-a}^{0}f(x)dx=2.\int_{0}^{a}f(x)dx$.

 

Vậy $I_1=0, I_2=2, I_3=0, I=1$.